Ở các phần tiếp theo việc xác định hệ số sẽ được trình bày một cách sơ lược bởi vì những bài toán đó mang tính phức tạp nhiều hơn mà U.C.T chỉ đơn thuần là bước đệm để đi đến lời giải [r]
(1)KỸ THUẬT HỆ SỐ BẤT ĐỊNH GIẢI BẤT ĐẲNG THỨC UCT WWW.TOANMATH.COM Nguyễn Thúc Vũ Hoàng Học sinh chuyên Toán-Tin-THPT Chuyên Lê Quí Đôn-Niên khóa 2006-2008 Thị xã Đông Hà-Tỉnh Quảng Trị Võ Quốc Bá Cẩn Sinh viên K32 Khoa Dược-Đại học Y Dược Cần Thơ -Niên Khóa 2006-2011 Thành Phố Cần Thơ Có bao nhiêu điều bí ẩn mà bạn chưa biết đến ?! Câu trả lời là rất nhiều và đôi bạn cảm thấy bực bội, khó chịu không thể tìm lời giải thích thỏa đáng cho bí ẩn nào đó Nhưng bạn hãy quan niệm đằng sau bất kì điều gì luôn hàm chứa ý nghĩa định Và không phải ngẫu nhiên mà lí giải lại hình thành Trong giới bất đẳng thức Đôi bạn không thể hiểu người ta lại có thể tìm lời giải trông có vẻ “kì cục” !!! Phải là lần mò và may rủi tìm ? Câu trả lời lại lần nữa nhắc lại: mỗi lời giải có giải thích riêng thân nó Việc tìm lời giải đó phải qua quá trình lập luận, thử, sai và đúng Trong chuyên đề nho nhỏ này chúng tôi muốn giới thiệu đến các bạn kĩ thuật không kém phần hiệu việc chứng minh số dạng bất đẳng thức Nó không giúp ta giải tất các bài toán mà giúp ta tìm những lời giải ngắn gọn và ấn tượng lớp bài toán nào đó Một số bài toán dễ đối với phương pháp này lại là khó đối với kỹ thuật Đây là điều hiển nhiên và dễ hiểu Mục lục Phần Phần Phần Phần Phần Phần Phần Phần Phần Phần Bài toán mở đầu Khởi đầu cùng số bài toán Kĩ thuật chuẩn hóa và U.C.T U.C.T và kỹ thuật phân tách các trường hợp Kết hợp bất đẳng thức Vornicu Schur với U.C.T Một dạng biểu diễn thú vị Giải số bài toán mà điều kiện liên quan mật thiết đến U.C.T mở rộng Lời kết 10 Bài tập áp dụng Tìm tài liệu Toán ? Chuyện nhỏ - www.toanmath.com pg (2) Phần Bài toán mở đầu Bài toán [Nguyễn Thúc Vũ Hoàng] Cho a, b, c là các số thực dương thỏa mãn a b c Chứng minh 1 2(a b c ) 5 a b2 c2 Chứng minh Ta sử dụng bất đẳng thức sau đây 2a 2a a2 3 Thật bất đẳng thức trên tương đương với (a 1) (2a 6a 3) 0 3a Hiển nhiên đúng với a là số thực dương Sử dụng các bất đẳng thức tương tự với b và c Ta có điều phải chứng minh Đẳng thức xảy a b c Chắc chắn đọc lời giải cho bài toán “ đơn giản” này bạn có phần lúng túng và không hiểu lại có thể tìm bất đẳng thức phụ cách “khó hiểu” Phải là dự đoán cách “vô hướng” Hoặc có người nghĩ bài toán trên tạo từ chính bất đẳng thức phụ đó Câu trả lời là hoàn toàn không phải Tất theo qui luật nó Ở các phần chúng tôi phân tích kỹ thuật phân tích giúp tìm các bất đẳng thức phụ và mở rộng vấn đề này theo chiều hướng khá mẻ Kỹ thuật này có tên là U.C.T, là viết tắt chữ cái đầu cụm từ tiếng Anh Undefined Coefficient Technique Hay còn gọi là Kỹ Thuật Hệ số bất định Đây là kỹ thuật và là tảng quan trọng trên đường tìm kiếm lời giải cho những bất đẳng thức khó Phần Khởi đầu cùng một số bài toán bản Chúng ta khởi đầu kỹ thuật này việc đưa cách giải thích cho việc tìm bất đẳng thức phụ trên và nó chính là cách giải thích cho các bài toán sau này chúng ta Bài toán trên các biến vế và điều kiện không ràng buộc điều này khiến ta nghĩ tách theo biến để chứng minh đơn giản có thể Nhưng rõ ràng ta đó thôi là không đủ Nếu ta chứng minh bất đẳng thức sau 2a (a 1)( a 1)( 2a 3) 0 3 3a a2 Rõ ràng không hoàn toàn đúng với a thực dương Đừng bỏ đây vì cách trên ta chưa sử dụng điều kiện a b c Như ta không theo đường lối suy nghĩ đơn giản ban đầu nữa mà tìm hệ số để bất đẳng thức sau là đúng 2a ma n (1) 3 a2 Trong đó m và n là các hệ số chưa xác định Tương tự với biến b và c Cộng vế theo vế ta có Tìm tài liệu Toán ? Chuyện nhỏ - www.toanmath.com pg (3) 1 2a 2b 2c 5 m(a b c) 3n 3(m n) 2 3 a b c Như đây hệ số m và n phải thỏa mãn điều kiện m n n m Thế vào (1) dẫn đến 2a m(a 1) (2) 3 a2 Đến đây ta cần xác định hệ số là m để bất đẳng thức (2) là đúng Chú ý bài toán này điểm cực trị đạt a b c nên ta cần xác định m cho (a 1)(2a 3) 2a m ( a ) ( a ) m 2 3 a 3a (a 1)( 2a 3) 2 từ đó ta dự đoán m để tạo 3 3a thành đại lượng bình phương (a 1) biểu thức Từ đó ta chứng minh bất đẳng thức phụ 2a 2a a2 3 Khi cho a thì ta có Quá trình tìm bất đẳng thức phụ đã phân tích cụ thể trên Tuy nhiên đó không phải là cách để ta tìm hệ số Ta có thể sử dụng tính chất đường tiếp tuyến điểm đồ thị hay sử dụng đạo hàm Nhưng có lẽ cách dự đoán trên là hữu hiệu và đơn giản mặt trực quan thực Tuy nhiên tất là dự đoán Nó không đảm bảo sau tìm bất đẳng thức phụ thì bài toán giải Một số dạng toán đề cập các phần chuyên đề này Ở phần này chúng ta chứng minh số bất đẳng thức đề hình thành đầu kỹ thuật qua đó thành thục việc phân tích Ta tiếp tục đến với bài toán sau Bài toán [Vasile Cirtoaje] Cho a, b, c, d là các số thực dương thỏa mãn a b c d Chứng minh 1 1 2 a 1 b 1 c 1 d 1 Chứng minh Ta xác định hệ số m để bất đẳng thức sau là đúng (a 1)(a 1) a 1 m(a 1) m(a 1) (a 1) m 2 a 1 a 1 a 1 a 1 1 m 1 Ta dự đoán bất đẳng thức sau đúng và thật Khi a ta có a 1 a (a 1) 2 a 0 a2 1 a2 1 Tương tự với các biến còn lại Cộng vế theo vế ta có điều phải chứng minh Đẳng thức xảy và a b c d Nhận xét Ta có thể sử dụng kỹ thuật “Côsi ngược dấu” để tìm bất đẳng thức phụ trên Tìm tài liệu Toán ? Chuyện nhỏ - www.toanmath.com pg (4) a2 a2 a 1 1 2 2a a 1 a 1 Bài toán [Algebraic Inequalities Old and New Method] Cho a, b, c là các số thực dương thỏa mãn a b c Chứng minh 1 1 a bc b ca c ab Chứng minh Ở đây ta cần tìm m để bất đẳng thức là đúng 1 a(a 1) m(a 1) m(a 1) a bc a a3 3(a a 3) Tương tự trên ta tìm dự đoán với m thì bất đẳng thức phụ đúng Thật a (a 1) (3 a) (a 1) (b c) a2 a 9 3(a a 3) 3(a a 3) Nhận xét Bài toán trên có thể giải kĩ thuật “Phân tách Chebyshev” xem cách giải U.C.T lại đơn giản mặt ý tưởng Bài toán tổng quát đã giải định lí LCF “Algebraic Inequalities Old and New method” tác giả Vasile Cirtoaje Cho a1 , a2 , , an là các số thực không âm thỏa mãn a1 a2 an n Chứng minh 1 1 a1 a1 n a2 a2 n an an n Bài toán [Nguyễn Thúc Vũ Hoàng] Cho a, b, c, d là các số thực không âm thỏa a b c d Chứng minh 2(a b c d ) ab ac ad bc bd dc Chứng minh Theo bài a, b, c, d là các số thực dương thỏa mãn a b2 c2 d (a b c d ) 2(2 ab ac ad bc bd cd ) (a b c d ) 2(2 ab ac ad bc bd cd ) Bất đẳng thức cần chứng minh tương đương với 2(a b c d ) (a b c d ) Ta cần xác định hệ số m để bất đẳng thức sau đúng 3a (2a 1) (a 1) 2a m(a 1) m(a 1) 2 Dễ dàng dự đoán m Ta chứng minh điều đó, 3a 9(a 1) 2a 2(a 1) (a 2) 2 Điều này hiển nhiên đúng Đẳng thức xảy và a b c d Tìm tài liệu Toán ? Chuyện nhỏ - www.toanmath.com pg (5) Nhận xét Bài toán này với hình thức khá “cồng kềnh” vì chứa thức Tuy nhiên nhận điểm mấu chốt bài toán ta dễ dàng đưa đơn lượng theo biến để giải Bài toán trên còn có thể giải theo cách khác cách chứng minh trực tiếp với biến Nhưng dù việc giải theo biến riêng biệt dễ dàng nhiều Bài toán Cho a, b, c là các số thực dương thỏa mãn a b c Chứng minh 1 1 4 5(a b c ) 27 a b c Chứng minh Ta cần tìm hệ số m cho (a 1)(5a 5a 4) 5a m(a 1) m(a 1)(a a 1) a a Ta dễ dàng nhận đẳng thức xảy và a b c Khi cho a thì ta có thể dự đoán m Ta chứng minh với m thì bất đẳng thức phụ trên là đúng Thật (a 1) (2a a 4) 5a 2a 0 a a Do a 3 2a a Vậy bất đẳng thức phụ trên là đúng Đẳng thức xảy và a b c Bài toán Cho a1 , a2 , , an là các số thực không âm thỏa mãn n a i 1 n 3a i 1 i n Chứng minh n i 5 Chứng minh Ta tìm hệ số m cho (5 3ai )(ai 1) m(ai 1) m(ai 1) 3ai 8(3ai2 5) Ta dự đoán với m thì bất đẳng thức phụ trên là đúng Thật vậy: 32 (5 )(ai 1) (ai 1) 0 32 3ai2 32(3ai2 5) Điều này hiển nhiên đúng Đẳng thức xảy và các biến và Nhận xét Qua các bài toán trên ta có thể thấy bất đẳng thức không quan tâm đến số biến Ta hoàn toàn có thể tổng quát với n biến mà không làm ảnh hưởng đến cách giải Đây là điểm thú vị U.C.T Một cách tổng quát ta đưa cách giải cho lớp bài toán có dạng sau Bài toán tổng quát Cho các số thực không âm a1 , a2 , , an thỏa mãn h(a1 ) h(a2 ) h(an ) Chứng minh f (a1 ) f (a2 ) f (an ) Tìm tài liệu Toán ? Chuyện nhỏ - www.toanmath.com pg (6) Lớp bài toán này có thể giải cách phân tách để chứng minh theo biến Vì các biểu thức mang tính đối xứng với nên thường thì điểm cực trị đạt các biến Ta phải xác định hệ số m cho f (ai ) m h(ai ) Đúng với biến thỏa mãn điều kiện đặt Với cách giải này ta giải lượng lớn các bất đẳng thức mà các biến không ràng buộc lẫn cách “mật thiết” n Thường là số dạng điệu kiện aik n Có thể khái quát tư tưởng kỹ thuật i 1 này lớp bài toán trên sau: Để chứng minh bài toán ta xác định hệ số các bất đẳng thức phụ theo biến riêng biệt cho f (ai ) m h(ai ) g (ai )2 k p(ai ) Trong đó g (ai ) (ai xk ) với x k là điểm cực trị bất đẳng thức Bài toán giải p(ai ) Trong trường hợp p(ai ) đúng miền nghiệm nào đó thì ta tiến hành chia trường hợp để giải bài toán Tuy nhiên phần này ta không đề cấp đến những bài toán mà đề cập phần sau Sau đã tìm bất đẳng thức phụ Với nhiều công cụ đạo hàm, khảo sát hàm số hay đơn giản là phân tích nhân tử ta có thể giải không quá khó khăn Trong phép chứng minh cho các bất đẳng thức phụ trên ta biến đổi và qui việc phân tích nhân tử đa thức an x n an1 x n1 a2 x a1 x a0 Mà mục đích chủ đạo là qui dạng tổng các bình phương Việc nhân tích đa thức thành nhân tử là vấn đề Đại số nên xin không nêu đây Qua vài ví dụ nho nhỏ hẳn phần nào các bạn đã hiểu U.C.T Ở các phần việc xác định hệ số trình bày cách sơ lược vì những bài toán đó mang tính phức tạp nhiều mà U.C.T đơn là bước đệm để đến lời giải không thể đưa ta cách chứng minh trực tiếp Phần Kĩ thuật chuẩn hóa và U.C.T Bây chúng ta bước sang khoảng không gian với lớp bất đẳng thức đối xứng ba biến và kĩ thuật chuẩn hóa kết hợp với U.C.T Đa thức f (a, b, c) đối xứng định nghĩa dạng: f (a, b, c) f / (a / , b / , c / ) đó (a / , b / , c / ) là hoán vị tùy ý (a, b, c) Hay nói cách khác là f (a, b, c) f (b, c, a) f (c, a, b) Tính đa thức đối xứng ba biến trên miền D có nghĩa là f (ka, kb, kc) k n f (a, b, c) với k , a, b, c D, n const phụ thuộc vào hàm f (a, b, c) Hiểu cách đơn giản đa thức nó là tổng các đơn thức đồng bậc Do số tính chất hàm ta có thể chuẩn hóa điều kiện biến để đơn giản hóa việc chứng minh Ta có thể chuẩn hóa đa thức đối xứng ba biến cách đặt a n b n c n k , abc p, ab bc ca r , Đây là kỹ thuật quan trọng giúp ta đơn giản hóa và qui bất đẳng thức chứng minh theo biến Hãy cùng đến với số bất đẳng thức đối xứng ba biến để thấy công dụng U.C.T Tìm tài liệu Toán ? Chuyện nhỏ - www.toanmath.com pg (7) Bài toán [Bất đẳng thức Nesbit] Cho a, b, c là các số thực không âm Chứng minh a b c bc ca ab Chứng minh Không tính tổng quát chuẩn hóa a b c Bài toán qui việc chứng minh a b c 3 a 3b 3c Ta cần chứng minh bất đẳng thức a 3(a 1) m(a 1) m(a 1) 3 a 2(3 a) Dễ dàng dự đoán m Ta chứng minh bất đẳng thức với m thì luôn đúng a 3a 3(a 1) 0 3 a 4(3 a) Điều này hiển nhiên đúng Sử dụng tương tự với các biến còn lại Cộng vế theo vế ta có điều phải chứng minh Đẳng thức xảy a b c Nhận xét bất đẳng thức Nesbit là bất đẳng thức đại số và có nhiều phép chứng minh Lời giải trên là lời giải đẹp và ngắn gọn cho bất đẳng thức này Bài toán [Võ Quốc Bá Cẩn] Cho a, b, c là các số thực không âm Chứng minh (b c a) ( a c b) (a b c) 3(a b c ) 2a (b c) 2b (a c) 2c (b a) (a b c) Chứng minh Chuẩn hóa a b c Khi đó bất đẳng thức cần chứng minh tương đương với 2(3 2a) 2(3 2b) 2(3 2c) a2 b2 c2 2 a 2a b 2b c 2c Ta cần xác định hệ số m để bất đẳng thức sau là đúng 2(3 2a) a m(a 1) a 2a Ta lại có 2(3 2a) (a 1)(a 3)( a 4a 6) a a 2a a 2a Từ đây dễ dàng dự đoán với m 6 thì bất đẳng thức phụ trên là đúng Thật 2(3 2a) (a 1) (6 a) a a 6( a 1) 0 a 2a a 2a Điều này hiển nhiên đúng a (0,3) Tương tự với các biến còn lại Đẳng thức xảy và a b c Bài toán [Đề thi Olympic 30-4, khối 11, lần XII – 2006] Cho a, b, c là các số thực dương Chứng minh a(b c) b (c a ) c ( a b) 2 2 2 (b c) a (c a ) b ( a b) c Tìm tài liệu Toán ? Chuyện nhỏ - www.toanmath.com pg (8) Chứng minh Không tính tổng quát, chuẩn hóa a b c Ta có bất đẳng thức cần chứng minh tương đương với a(3 a) b(3 b) c(3 c) 2 6a 2a 6b 2b 6c 2c Tương tự trên ta dễ dàng tìm bất đẳng thức phụ sau: a (3 a ) 21 9a (a 1) (18a 9) 6a 2a 25 25(9 6a 2a ) Điều này hiển nhiên đúng Đẳng thức xảy và a b c Nhận xét Có thể thấy hai lời giải cho các bài toán mở đầu phần đơn giản và ngắn gọn Đây có thể xem là kỹ thuật chính thống Giúp ta giải số bài toán “cùng loại” và đã quen thuộc sau Bài toán [Darij Grinberg, Old and New Inequalities] Cho a, b, c là các số thực dương Chứng minh a b c 2 (b c) (c a ) (a b) 4(a b c) Chứng minh Không tính tổng quát, giả sử a b c Bài toán cần chứng minh qui dạng sau a b c 2 (3 a) (3 b) (3 c) Dễ dàng dự đoán bất đẳng thức phụ sau a 2a (a 1) (9 2a ) 0 (3 a ) 4(3 a) Điều này hiển nhiên đúng a [0,3) Sử dụng bất đẳng thức này cho b, c cộng lại, ta có đpcm Bài toán 10 [Phạm Văn Thuận, Mathlinks forum] Cho a, b, c là các số thực dương Chứng minh (b c 3a ) (a c 3b) (a b 3c) 2a (b c) 2b (a c) 2c (b a ) 2 Chứng minh Không tính tổng quát, chuẩn hóa a b c Ta có bất đẳng thức cần chứng minh tương đương với (3 4a ) (3 4b) (3 4c) 2 2 2 2a (3 a ) 2b (3 b) 2c (3 c) Sử dụng bất đẳng thức phụ sau (3 4a) 8a ( a 1) (39 8a) 0 2a (3 a) 6(a 2a 3) Điều này hiển nhiên đúng vì a 39 8a 39 24 15 Tương tự với các biến còn lại ta có điều phải chứng minh Đẳng thức xảy và a b c Bài toán 11: [USAMO 2003] Cho a, b, c là các số thực dương Chứng minh (b c 2a ) (a c 2b) ( a b 2c ) 8 2a (b c) 2b (a c) 2c (b a) Tìm tài liệu Toán ? Chuyện nhỏ - www.toanmath.com pg (9) Chứng minh Không tính tổng quát, chuẩn hóa a b c Khi đó ta có bất đẳng thức cần chứng minh tương đương với (a 1) (b 1) (c 1) 8 2a (1 a ) 2b (1 b) 2c (1 c) Sử dụng bất đẳng thức phụ sau (a 1) 12a (3a 1) (4a 1) 2a (1 a ) 2a (1 a ) Điều này hiển nhiên đúng Đẳng thức xảy và a b c Phần U.C.T và kỹ thuật phân tách các trường hợp Ở các phần trên ta đã làm quen với số bài toán đưa dạng f (ai ) m h(ai ) g (ai )2k p(ai ) Thì có điều phải chứng minh Tuy nhiên không phải nó xuất p(ai ) Trong trường hợp p(ai ) đúng với miền nghiệm nào đó thì việc chứng minh phải qua chiều hướng khác, đó là xét thêm trường hợp biến ngoài miền xác định để p(ai ) Thường thì bước này phức tạp và đòi hỏi người làm phải có những đánh giá mang tinh tế nhiều Chúng ta đến với số bài toán tiêu biểu cho kỹ thuật này Bài toán 12 Cho a, b, c là các số thực dương Chứng minh a2 b2 c2 2 2 2 a (b c) b (a c) c (b a ) Chứng minh Không tính tổng quát chuẩn hóa a b c Qui bất đẳng thức dạng 3 a2 b2 c2 a2 2 2 2 5 a (3 a) b (3 b) c (3 c) cyc 2a 6a Ta sử dụng bất đẳng thức phụ sau a2 12a (8a 21)(a 1) 2a 6a 25 Không tính tổng quát giả sử a b c a c Xét hai trường hợp sau 21 8a 21 8b 21 8c 21 + Trường hợp c 21 + Trường hợp max{a, b, c} Khi đó ta có: a2 49 f (a) 2a 6a 50 3 1 a Do f (a) đồng biến trên (0,3] nên điều này hiển nhiên đúng Vậy bài toán chứng minh Đẳng thức xảy và ba biến Tìm tài liệu Toán ? Chuyện nhỏ - www.toanmath.com pg (10) Bài toán 13 [Vasile Cirtoaje - Algebraic Inequalities – Old and New Method] Cho a, b, c, d là các số thực dương thỏa mãn a b c d , Chứng minh 1 1 16 3a 3b 3c 3d Chứng minh Ta cần xác định hệ số để bất đẳng thức sau là đúng m(2a 1) 3a Dễ dàng tìm bất đẳng thức phụ sau 52 48a 3(2a 1) (12a 1) 0 3a 49 49(3a 1) Tương tự với các biến còn lại Xét hai trường hợp sau đây + Trường hợp 1 min{a, b, c, d } 12a 12b 12c 12d 12 + Trường hợp 49 48 d 3d 12 48 3d 49 Xét tương tự với các biến còn lại ta tìm điều phải chứng minh Đẳng thức xảy và a b c d Bài toán 14 [Vasile Cirtoaje, Algebraic Inequalities – Old and New Method] Cho a, b, c là các số thực dương thỏa mãn a b2 c Chứng minh a5 a b5 b c5 c 0 a b c b5 a c c b a Chứng minh Bất đẳng thức trên tương đương với 1 2 2 2 a b c b a c c b a a b2 c2 Từ đây suy ta cần chứng minh trường hợp a b2 c là đủ Áp dụng bất đẳng thức AM-GM ta có 2a 2a a5 2 a 1 a Đặt a x, b2 y, c z lúc đó ta có x y z và đó ta phải chứng minh 1 1 3 2x 2y 2z x3 y3 z 3 x 1 y 1 z 1 x 1 y 1 z 1 1 2 2x x 2x y y y 2z z 2z x 1 3 x 0 2x x 2x cyc ( x 1) (2 x x 0 cyc 6(2 x x x 3) Tìm tài liệu Toán ? Chuyện nhỏ - www.toanmath.com pg 10 (11) Không tính tổng quát giả sử x y z x z Xét hai trường hợp + Trường hợp y z x đó ta có 2 x 3x 0, 2 y y 0, 2 z 3z Dẫn đến bài toán hiển nhiên đúng + Trường hợp y z x đó ta có 2 (2 x3 x x 3) 5( x 1) x3 x 3x x x x x x3 x 2 0 2 x 1 ta cần chứng minh 2x x 2x z 1 y 1 2 2z z 2z y y y Điều này luôn luôn đúng vì với k 0,1 ta có Từ đó suy k 1 4k (k 1)(2k 1) 2k k 2k thì bài toán giải Nếu k thì ta có 4k (k 1)(2k 1) 4k 2(2k 1) 2(2k 2k 1) Nếu k Từ y z y, z 0,1 2(k 2k 1) 2(k 1) Vậy bài toán giải hoàn toàn Đẳng thức xảy và a b c Nhận xét Đây là kết “mạnh hơn” cho bài toán kì thi IMO 2005 tác giả Vasile Cirtoaje Bài toán gốc ban đầu là với điều kiện abc Điều kiện bài toán trên chặt vì theo bất đẳng thức AM-GM ta có a b c 3 (abc) abc Chúng ta hãy đến với lời giải chính tác giả bài toán trên, trích từ “Algebraic Inequalities, Old and New Method” Ta qui việc chứng minh bài toán sau: Cho a, b, c là các số thực dương thỏa mãn a b2 c Chứng minh 1 1 2 a a b b c c2 Không tính tổng quát ta giả sử a b c Xét hai trường hợp sau + Trường hợp a a, b Ta sử dụng các bất đẳng thức phụ sau a2 b2 c2 , , a5 a b b2 c5 c Lại có a (a 1) (a 2a 3a 6a 3) a5 a 6(a a ) Mặt khác Tìm tài liệu Toán ? Chuyện nhỏ - www.toanmath.com pg 11 (12) a 2a 3a 6a a a 2a a a 3 1 a 2 a 2 2 2 2 + Trường hợp a 2, a b c b c đó ta có 1 1 1 2 2 a 3 a b 3b c 3c a 3 a b c2 Lại có 1 1 2 2 a 3 a 2a a (2 1)a (2 1)2 Như bài toán giải 1 2 3b 3c Thật 1 9(b c 1) 5b c 0 b2 c2 6(3 b )(3 c ) Như bài toán giải Đẳng thức xảy và a b c Lời giải tác giả Vasile Cirtoaje từ đầu đã sử dụng U.C.T nó lại đưa ta đến cách xét trường hợp khá lẻ vì phải so sánh biến với Đây là bài toán đẹp với nhiều mở rộng thú vị Bài toán 15 [Võ Quốc Bá Cẩn] Tìm số k tốt để bất đẳng thức sau đúng với a, b, c 0 a3 b3 c3 3(a b c) 2 2 2 ka (b c) kb (c a) kc (a b) k 4 Chứng minh Cho a b 1, c ta k Ta chứng minh chính là giá trị cần tìm, tức là qui chứng minh a3 b3 c3 (a b c) 2 2 2 5a (b c) 5b (c a) 5c (a b) Sử dụng bất đẳng thức Cauchy-Schwarz, ta có a3 2 cyc 5a (b c) Ta cần chứng minh a2 (a b c) 2 cyc 5a (b c) a2 2 cyc 5a (b c ) Không tính tổng quát ta chuẩn hóa a b c và a b c suy a c0 Bất đẳng thức cần chứng minh tương đương với a2 b2 c2 2 6a 2a 6b 2b 6c 2c Ta phải xét hai trường hợp + Trường hợp c ta có Tìm tài liệu Toán ? Chuyện nhỏ - www.toanmath.com pg 12 (13) 9 cyc 27a 27a (3a 1) (8a 1) 12 a 0 6a 2a cyc cyc 6a 2a + Trường hợp c ta có 6a 6b 6c 2a 2b 6c 2 6a 2a 6b 2b 6c 2c 6a 2a 6b 2b 6c 2c a bc bca 6c 6a 2a 6b 2b 6c 2c 2(a b) (3c 2) 6c 1 c 2 (6a 2a 1)(6b 2b 1) 6c 2c 6a 2a 6b 2b Ta cần chứng minh 6c 1 2 6c 2c 6a 2a 6b 2b 1 6c Vì c nên nên ta chứng minh bất đẳng thức sau 6c 2c 1 1 6a 2a 6b 2b 1 Nếu b đó 1 6b 2b 1 Nếu b , áp dụng bất đẳng thức Cauchy-Schwarz, ta cần chứng minh 6(a b2 ) 2(a b) Điều này tương đương với 2(a b) c (a b c) 3(a b2 ) 3b a đó 2(a b) c (a b c) 2(a b)2 3(a b ) 4ab a b Từ giả thiết b 3(a b ) a(3b a) 3(a b ) Như bài toán đã chứng minh Đẳng thức xảy và a b c a b, c và các hoán vị Hằng số k tốt cần tìm là Bài toán 16 [Nguyễn Văn Thạch] Cho các số dương a, b, c thỏa a b c 3, chứng minh bất đẳng thức 1 3 2 a 3a b 3b c 3c Chứng minh Không tính tổng quát, giả sử a b c Với x 1 , ta có 2 x 3x x 1 Thật vậy, bất đẳng thức tương đương Tìm tài liệu Toán ? Chuyện nhỏ - www.toanmath.com pg 13 (14) ( x 1)2 ( x x 1) (đú ng) Từ đây, suy 1 +, Nếu c , sử dụng bất đẳng thức trên, ta có đpcm 1 +, Nếu c , có khả xảy ++, Nếu b , ta có 3 3 a 3a a 2 4 b 3b (b 1)2 b 2 1 1 c 3c (1 c) c 1 2 2 16 1 Do đó VT 1 1 3 ++, Nếu b , suy a b , xét hàm số f ( x) x 3x với x , ta có x 24 x 15 0 4( x x 3)5/ Suy f ( x) là hàm lõm, đó theo bất đẳng thức Jensen, ab f (a ) f (b) f f (t ) t 3t Ta phải chứng minh 3 2 t 3t (3 2t ) 3(3 2t ) Hay 3 t 3t 4t 6t Hay 36(t 1) (36t 252t 749t 1202t 1099t 546t 117) 0 (t 3t 3) (4t 6t 3) Dễ dàng kiểm tra bất đẳng thức này đúng, ta có đpcm Đẳng thức xảy và a b c f // ( x) Bài toán 17 [Mở rộng từ Poland 1996] Cho a, b, c là các số thực thỏa mãn a b c Chứng minh a b c a b c 10 Chứng minh Không tính tổng quát giả sử a b c a c Xét hai trường hợp sau: Tìm tài liệu Toán ? Chuyện nhỏ - www.toanmath.com pg 14 (15) ta có a (3a 1)2 (4a 3) b c a 18a 0 10 a b c cyc 25 30 a cyc 50(a 1) + Ttrường hợp c áp dụng bất đẳng thức AM-GM, ta có a b 1 a 1 b 1 c Khi đó 5 c ta có điều phải chứng minh c 1 10 a Xét trường hợp: 5 c đó ta có a Từ đây suy ra: a 1 a b c a b 1 a b c a b 10 10 Vậy bất đẳng thức chứng minh Đẳng thức xảy và a b c Nhận xét Bài toán gốc đề toán này là với điều kiện trường hợp Tuy nhiên bài toán đúng với số thực, đây là điều lí thú Có thể chứng minh bài toán trên với kỹ thuật dồn biến hàm lồi + Trường hợp c Phần Kết hợp bất đẳng thức Vornicu Schur với U.C.T Trong phần này chúng tôi giới thiệu đến các bạn việc kết hợp U.C.T với bất đẳng thức Vornicu Schur Có thể nói ta kết hợp nhuần nhuyễn hai kỹ thuật trên thì nhận những lời giải khá ấn tượng và đẹp mắt Trước hết hãy cùng đến với dạng phát biểu, các định lí kỹ thuật phân tích chính tắc bất đẳng thức Vornicu Schur Bất đẳng thức Vornicu Schur: Cho a b c và A, B, C đó bất đẳng thức A(a b)(a c) B(b c)(b a) C (c a)(c b) Là đúng và chỉ Định lí A B C B Định lí A a B b Định lí B c C b (Nếu a,b,c là ba cạnh tam giác) Định lí A C B Khi đã nắm tay các định lí bất đẳng thức Vornicu Schur thì hẳn bạn phải chú ý đến cách biến đổi cho qui dạng chính tắc nó Ở đây xin nêu phép biến đổi cực kì hiệu và có công dụng lớn nhiều bài toán, giúp bạn có thể đưa bài toán từ dạng tổng các bình phương dạng trên Trước hết hãy biến đổi đưa bài toán hai dạng quen thuộc sau Dạng A(a b)2 B(b c)2 C (c a)2 Dạng A(2a b c)2 B(2b c a)2 C (2c a b)2 Tìm tài liệu Toán ? Chuyện nhỏ - www.toanmath.com pg 15 (16) Tiếp tục thực phép biến đổi sau A(a b)2 B(b c) C (c a) A(a b)(a c c b) B(b c)(b a a c) C (c a)(c b b a) A(a b)(a c) A(b c)(c a) ( A B)(a b)(a c) cyc cyc cyc Dạng là dạng phân tích chính tắc phương pháp S.O.S phương pháp đã lấy làm quen thuộc với nhiều người Từ phép phân tích trên ta có thể thấy mối liên hệ giữa phương pháp S.O.S và bất đẳng thức Vornicu Schur là mật thiết Tuy nhiên bài viết này không đề cập đến vấn đề này mà chúng ta xem xét dạng trên Vì tính ứng dụng nó U.C.T là nhiều và nó là kết hợp mang nhiều ý nghĩa A(2a b c) B(2b c a) C (2c a b) 2 A(a b)(a c) A(a b) A(c a) cyc cyc cyc 2 A(a b)(a c) ( A B)(a b) cyc cyc 2 A(a b)(a c) (2 A B C )(a b)(a c) cyc cyc 2 (4 A B C )(a b)(a c) cyc Hãy mở đầu bài toán trông có vẻ đơn giản không quá dễ để tìm lời giải không chọn đúng đường Bài toán 18 [Vasile Cirtoaje] Cho a, b, c là các số thực không âm thỏa mãn a b c Chứng minh 3(a b4 c ) a b2 c 6(a3 b3 c3 ) Chứng minh Theo U.C.T dễ dàng tìm bất đẳng thức phụ sau 3a a 3a3 4a (a 1)2 (3a 2) Ta qui bài toán chứng minh (a 1)2 (3a2 2) cyc Thật (a 1) (3a 2 2) cyc (3a 3) (3a 2) cyc (3a a b c) (3a 2) cyc (2a b c) (3a 2) cyc (4a b c 4)(a b)(a c) cyc Không tính tổng quát giả sử a b c đó ta có 4a b2 c2 4b2 a c 4c b2 a Lại có ( a b) (3 c) (3c 1) c a b 4c 4c 4 0 2 Tìm tài liệu Toán ? Chuyện nhỏ - www.toanmath.com pg 16 (17) Theo định lí ta có điều phải chứng minh Đẳng thức xảy và a b c 4 (a, b, c) , , 3 3 Nhận xét Bài toán giải trường hợp 3a a Trường rõ ràng khó giải vì vế phải điều kiện trường hợp khá lẻ, nhiều khả dẫn đến những tính toán lằng nhằng không cần thiết Tuy nhiên cần chú ý điều là đẳng thức bài toán này xảy hai điểm cực trị vì không thể áp dụng mỗi U.C.T vì dạng phát biểu kỹ thuật này cho ta điểm cực trị cần tìm Như việc kết hợp giữa U.C.T và bất đẳng thức Vornicu Schur không đơn là giải bài toán cách đẹp mắt mà còn hướng ta đến việc giải trường hợp đẳng thức xảy có hai biến và khác biến còn lại hợp còn lại a Bài toán 19 [Nguyễn Thúc Vũ Hoàng] Cho a, b, c là các số thực dương thỏa mãn a b c Chứng minh 2(a3 b3 c3 ) 5(a b2 c ) Chứng minh Ta cần xác định hệ số cho bất đẳng thức phụ sau: 2a3 5a m(a 1) (a 1)(2a 3a 3) m(a 1) Từ đây ta dự đoán m 4 ta có 2a3 5a 4a (a 1)2 (2a 1) Tương tự với các biến còn lại ta có bất đẳng thức cần chứng minh tương đương với 2(a b3 c3 ) 5(a b c ) (a 1) (2a 1) (b 1) (2b 1) (c 1) (2c 1) (2a b c) (2a 1) (2b c a) (2b 1) (2c a b) (2c 1) 6a(a b)(a c) 6b(b c)(b a ) 6c(c a )(c b) Không tính tổng quát giả sử a b c Khi đó theo bất đẳng thức Vornicu Schur ta có điều phải chứng minh Đẳng thức xảy và chi a b c a 0, b c cùng các hoán vị Nhận xét Lại bài toán đơn giản điều thú vị bài toán này là đẳng thức đạt điểm Nếu giải cách thông thường U.C.T thì không thể giải bài toán cách triệt để và lần nữa bất đẳng thức Vornicu Schur lại phát huy tác dụng nó Bài toán 20 [Vasile Cirtoaje, Romania TST 2006] Cho a, b, c là các số thực dương thỏa mãn a b c Chứng minh 1 a b2 c2 a b2 c2 Chứng minh Theo U.C.T dễ dàng tìm bất đẳng thức phụ sau 4a a a Bài toán cần chứng minh tương đương với Tìm tài liệu Toán ? Chuyện nhỏ - www.toanmath.com pg 17 (18) cyc (a 1) (1 2a a ) 0 a2 cyc (2a b c) (1 2a a ) 0 a2 (4 A B C )(a b)(a c ) cyc Trong đó 2a a 2b b 2c c , B , C a2 b2 c2 Không tính tổng quát giả sử a b c đó ta có 2b b 2a a ( a b)(2ab a b) A B 0 b2 a2 a 2b Từ đó suy 4C A B 4B A C A B C Áp dụng bất đẳng thức AM-GM ta có a b c 3 abc abc 3 abc Do đó 1 2 4A B C a b c a b c 1 6 1 1 3 a a b c a b c A 1 1 3 2(3 3) a b c abc Theo định lí ta có điều phải chứng minh Đẳng thức xảy và a b c Nhận xét Ở bài toán này chúng ta có thể chia trường hợp để giải Dưới đây là lời giải tác giả bài toán Vasile Cirtoaje Sau đã đưa bài toán dạng (a 1)2 (1 2a a ) 0 a2 cyc Không tính tổng quát giả sử a b c đó áp dụng định lí dấu tam thức bậc ta chia nhỏ bài toán thành hai trường hợp + Trường hợp a c b a từ đó dẫn đến 2a a 0,1 2b b2 0,1 2c c + Trường hợp a b c a suy (b c) bc Khi đó 1 1 2 2 2 18 (a b c) a b c 2 a b c a b bc Bài toán giải Đẳng thức xảy và a b c Còn nhiều lời giải các kỹ thuật khác cho bất đẳng thức trên Tuy nhiên khuôn khổ chuyên đề có hạn nên xin không nêu đây Tìm tài liệu Toán ? Chuyện nhỏ - www.toanmath.com pg 18 (19) Phần Một dạng biểu diễn thú vị Ở đây chúng tôi muốn nói đến dạng biểu diễn theo tổng Đây là tư tưởng đơn giản giúp ta tìm nhiều lời giải ấn tượng Bây ta hãy chú ý đến đẳng thức sau đây a k bk ck ak bk ck 1 k a bk ck a k bk c k a k bk c k a k bk c k Đẳng thức tưởng chừng là điều hiển nhiên, không mang nhiều ý nghĩa lại có vai trò khá quan trọng việc chứng minh lớp bất đẳng thức mà chúng tôi nêu đây Ở phần này kỹ thuật xác định hệ số không còn có thể thực trước vì đây xuất lũy thừa p Nếu sử dụng những biến đổi thông thường thì phức tạp Vì công cụ mà chúng ta chọn đây là đạo hàm Trước hết xin nhắc lại định lí sau đây Định lí Fermat Giả sử hàm số f ( x) xác định trên [a, b] và có cực trị địa phương x0 [a, b] Khi đó f có đạo hàm x0 thì f / ( x0 ) Định lí Roll Giả sử f :[a, b] liên tục và khả vi (a, b) Nếu f (a) f (b) thì tồn x0 (a, b) cho f / ( x0 ) Bài toán 21 [Võ Quốc Bá Cẩn] Tìm số k tốt để bất đẳng thức sau là đúng với số a, b, c là các số thực dương a b c 2 2 2 k 4 ka (b c) kb (c a) kc (a b) Chứng minh Cho a 1, b c ta có k Ta chứng minh đó là giá trị k tốt để bất đẳng thức là đúng Bất đẳng thức cần chứng minh a b c 1 2 2 a 2(b c) b 2(c a) kc (a b)2 Ta phải xác định hệ số k cho bất đẳng thức sau là đúng a ak k k k a 2(b c)2 a b c Ở đây ta chuẩn hóa b c để việc việc xác định hệ số đơn giản Khi đó ta cần xác định hệ số k cho a ak k a k 2a k a a 8 a k 2 2k Đặt f (a) a 2a a Lại có f (a) 0, f (1) nên theo định lí Fermat ta có f / (1) Tiến hành đạo hàm f (a) suy f / (a) (k 2)a k 1 4ka k 1 2a Theo trên thi ta có f / (1) (k 2) 4k k Như ta dự đoán bất đẳng thức sau là đúng Tìm tài liệu Toán ? Chuyện nhỏ - www.toanmath.com pg 19 (20) a a4 a 2(b c)2 a b4 c Sau đã hoàn thành xong bước dự đoán chúng ta có nhiều đường để lựa chọn Thông thường thì phép biến đổi tương đương luôn mang lại hiệu bất đẳng thức phụ là đúng Nên nhớ bất đẳng thức phụ trên là dự đoán mà thôi, có thể nó không đúng ngược lại Từng bài toán ta “tùy ứng biến” Tất nhiên nhiều bài toán không áp dụng theo cách này Chúng ta tiếp tục quay lại bài toán trên với phép chứng minh cho bất đẳng thức phụ bc b c 2 từ đây ta phải chứng minh Theo bất đẳng thức Holder ta có 4 bất đẳng thức a a 8t 3 a4 a4 t a t a a 8t t ( a t )2 bc Vậy bất đẳng thức này hiển nhiên đúng Đẳng thức xảy và a b c a t 0, b c và các hoán vị Nhận xét Quá trình tìm kiếm hệ số k có thể thông qua việc đánh giá theo bất đẳng thức AM-GM sau a ak k a k 2a k a a k a 2a k a 8 a Ở đây t Mặt khác theo bất đẳng thức AM-GM thì a k a a k Như ta có cần xác định k cho a k a k a k a k k 4k k Bài toán 22 [IMO 2001] Cho a, b, c là các số thực dương Chứng minh a b c 1 2 a 8bc b 8ca c 8ab Chứng minh Bằng cách làm tương tự, ta thiết lập bất đẳng thức sau a a4/ 4/3 4/3 4/3 a 8bc a b c Thật vậy, sử dụng bất đẳng thức AM-GM, ta có b4/ c4/ 2b2/ 3c2/ 2t 4/ , ta cần chứng minh a / 2t / a1/ a 8t 4t / (a / t / ) (đú ng) Do đó, bất đẳng thức trên đúng Sử dụng tương tự cho b, c cộng lại, ta có đpcm Đẳng thức xảy và a b c b 0, c Tìm tài liệu Toán ? Chuyện nhỏ - www.toanmath.com pg 20 (21) Bài toán 23 Cho a, b, c là các số thực không âm Chứng minh a3 b3 c3 1 a3 (b c)3 b (c a ) c ( a b) Chứng minh Tương tự trên ta có xác định bất đẳng thức phụ sau: a3 a2 (*) a (b c)3 a b c Có thể chứng minh bất đẳng thức phụ trên theo nhiều cách: Cách (*) 2a (b2 c ) (b2 c )2 a(b c)3 Điều này hiển nhiên đúng, 2a (b c ) (b c ) a (b c) (b c) a (b c)6 2 a(b c)3 4 Cách Theo bất đẳng thức AM-GM ta có (1 k ) (1 k k ) k2 k (1 k )(1 k k ) 1 2 Áp dụng bất đẳng thức phụ trên ta có a3 a3 (b c)3 a2 2 2 2 a b c b c 1 b c bc 1 a 2 a a Áp dụng tương tự với các biến còn lại Cộng vế theo vế ta có có điều phải chứng minh Đẳng thức xảy và biến có biến dần 1 Bài toán 24 Cho a, b, c là các số thực dương Chứng minh a3 b3 c3 3 3 3 a (b c) b (c a ) c (a b) Chứng minh Sử dụng bất đẳng thức Cauchy-Schwarz, ta có 1 a3 VT 3 cyc a (b c)3 Bất đẳng thức chứng minh Đẳng thức xảy và a b c Phần Giải một số bài toán mà điều kiện liên quan mật thiết đến Đa phần các bài toán xét đến trên có điều kiện mà các biến liên hệ với ko quá chặt Thường là điều kiện dạng a1k a2k ank1 ank n Tức là ta có thể tách theo biến để tìm bất đẳng thức phụ Tuy nhiên với số bài toán mà điều kiện thiết lập k n mối quan hệ “bền chặt” thì việc tìm bất đẳng thức phụ tương đối i 1 khó khăn vì ta không thể đánh giá theo biến nữa Và để áp dụng U.C.T những bài toán chúng ta phải dùng đến số tính chất hàm số Tìm tài liệu Toán ? Chuyện nhỏ - www.toanmath.com pg 21 (22) Bài toán 25 Cho a, b, c là các số thực dương thỏa mãn abc Chứng minh a bc b c a c a b b c 1 c a 1 a b 1 Chứng minh Áp dụng bất đẳng thức Holder ta có a b c b c a c a b a (b c 1) (a b c) bc b c c a a b cyc Do đó ta cần phải chứng minh a(b c 1) ( a b c ) 2 bc cyc a b a a3 3 3 6 4 ab 4 a 2 cyc cyc b cyc a cyc cyc cyc b c Áp dụng bất đẳng thức AM-GM ta có a b a a b ab, ab, 2 cyc b cyc a cyc b cyc cyc a cyc cyc b c Từ đó ta có a b VT VP a 4 ab 4 a 6 cyc b cyc a cyc cyc cyc a ab 4 a 6 a 4a a cyc cyc cyc cyc Xét hàm số f ( x) x3 x ln x với x ta có x 1 f / ( x) ( x 1) 3x x x 1 Nếu x thì , x đó f / ( x) x x x x Từ đó đễ dàng kiểm tra f ( x) f (1) 0, x Hay x3 x 2 ln x, x x Như ta có a 4a 2 ln a a cyc cyc Bài toán giải Đẳng thức xảy và a b c Bài toán 26 [Lê Hữu Điền Khuê, THPT Quốc Học, Thành phố Huế] Cho a, b, c là các số thực dương thỏa mãn abc Chứng minh 1 1 2 3a (a 1) 3b (b 1) 3c (c 1) Chứng minh Xét hai trường hợp sau Tìm tài liệu Toán ? Chuyện nhỏ - www.toanmath.com pg 22 (23) + Trường hợp Nếu ba số a, b, c tồn ít số không lớn Giả sử 3a (a 1) Khi đó bất đẳng thức hiển nhiên đúng + Trường hợp Cả ba số a, b, c không nhỏ đó ta xét hàm số sau Giống các phần trước ta có thiết lập bất đẳng thức phụ dạng 1 k ln x 2 x ( x 1) Ở đây ta có qui hàm số mũ và chú ý ln x ln y ln z Tiếp tục quan sát thấy đẳng thức xảy và a b c Từ đó ta có phải xác định k cho f / (1) f ( x) ln x x ( x 1) 3 Với x Khi đó ta có 2(16 x 16 x x 1) 2( x 1)(16 x 1) f / ( x) x(4 x x 1) x(4 x x 1) Từ đây suy f / ( x) x 1, x Dễ dàng kiểm tra f ( x) f (1) 0, x Điều này tương đương với 1 ln x, x 2 3x ( x 1) 3 Sử dụng bất đẳng thức phụ trên theo biến a, b, c cộng vế theo vế ta có 1 ln a 2 2 3a (a 1) 3b (b 1) 3c (c 1) cyc Bất đẳng thức chứng minh Đẳng thức xảy và a b c , a , b , c 0 và các hoán vị Nhận xét Bài toán trên còn lời giải ấn tượng Vasile Cirtoaje Xin trình bày lại lời giải đó Sử dụng bất đẳng thức phụ sau đây 1 2a( a 1) 0 3a (a 1) 2a (4a 2a 1)(2a 1) Điều này hiển nhiên đúng với số thực không âm Tương tự với các biến còn lại suy điều phải chứng minh số đó là a Ta có a Bài toán 27 [Gabriel Dospinescu] Cho a1 , a2 , , an là các số thực dương thỏa mãn a1a2 an Chứng minh a12 a22 an2 2(a1 a2 an ) Chứng minh Xét hàm số sau với x f ( x) x x ln x 2 Khi đó ta có Tìm tài liệu Toán ? Chuyện nhỏ - www.toanmath.com pg 23 (24) ( x 1) 2 x x x 2( x 1) f / ( x) x f ( x) 2 x 2( x 1)( x x 1) / Qua thì f / ( x) đổi dấu từ dương sang âm nên f / ( x) f (1) 0, x Điều đó có nghĩa là x2 x ln x, x 2 Sử dụng bất đẳng thức phụ này cho n biến và cộng vế theo vế ta có a12 a22 an2 2(a1 a2 an ) (ln a1 ln a2 ln an ) 2 n 2(a1 a2 an ) ln i 1 2(a1 a2 an ) Vậy bất đẳng thức chứng minh Đẳng thức xảy và a1 an Nhận xét Bài toán trên còn có thể giải bất đẳng thức phụ quen thuộc x2 2( x x 1) ( x 1)4 , x Sử dụng bất đẳng thức trên cho n biến cộng lại ta có n a12 a22 an2 2(a1 a2 an ) n i 1 2(a1 a2 an ) Bất đẳng thức đã giải hoàn toàn Bài toán 28 [Algebraic Inequalities – Old and New Method] Cho a, b, c là các số thực dương thỏa mãn abc Chứng minh a b2 c 9(ab bc ca) 10(a b c) Chứng minh Ta có cần xác định hệ số k cho bất đẳng thức sau là đúng a 9bc a 10a k ln a a Tương tự các phần trước ta có tìm k 17 Ta có chứng minh f (a) a 10a 17 ln a a Thật 17 2a 10a 17 a (a 1)(2a 8a 9) f / (a ) 2a 10 a a a2 a2 f / (a) a Từ đây, ta có thể dễ dàng thấy f (a) f (1) 0, a hay a 10a 17 ln a a Sử dụng tương tự với b, c cộng lại vế theo vế, ta có đpcm Đẳng thức xảy và a b c Tìm tài liệu Toán ? Chuyện nhỏ - www.toanmath.com pg 24 (25) Phần U.C.T mở rộng Ngay từ đầu bài viết ta đã xét đến việc xác định hệ số m theo cách h(ai ) f (ai ) mak n Với điều kiện xác định bài toán là a1k a2k ank n Tuy nhiên với cách xác định đó đối với số bài toán lại không mang lại hiểu Điều đó không phải hoàn toàn là không tốt Vì nó thôi thúc chúng ta tìm các dạng xác định hệ số khác Một cách trực quan chúng ta phân tích bài toán cụ thể để thấy những gì đã nêu trên Bài toán 29 [Tạp chí Crux, Canada] Cho a, b, c là các số thực dương thỏa mãn a b c Chứng minh 1 ab bc ac Chắc hẳn từ đầu vào chứng minh bài toán này bạn nghĩ đến việc thiết lập bất đẳng thức phụ dạng 8 mx n m( x 1) 9 x 9 x Dễ dàng dự đoán m Nhưng đáng tiếc với m thì bất đẳng thức trên hoàn toàn không đúng kể tư tưởng chia trường hợp phần không thể áp dụng Thật 7 x ( x 1) 0 9 x 8(9 x) Tuy nhiên U.C.T có tác dụng trường hợp này ý tưởng mẻ Hãy chú ý đến cách thiết lập bất đẳng thức phụ sau m( x 1) n( x 1) (*) 9 x Việc xác định hệ số bất đẳng thức trên đòi hỏi chặt chẽ lập luận vì đôi nới lỏng miền nghiệm biến khiến cho bài toán không đúng Có nhiều hệ số thỏa mãn để tạo thành đại lượng bình phương ( x 1)2 ta phải xác định cho dấu bất đẳng thức là đúng Ta có (*) ( x 1) m( x 1) n (**) 9 x Từ phân tích trên rõ ràng ta phải xác định n theo m cho xuất nghiệm x để hình thành đại lượng ( x 1)2 , tức là 1 m( x 1) n 0n m( x 1) n 2m 9 x 9 x Từ đây vào (**) ta có 1 (**) ( x 1) m( x 1) 2m 9 x ( x 1) (72m 8mx 1) Dễ thấy việc xác định hệ số đây không còn đơn giản trước Nó đòi hỏi ta phải tìm những ước lượng chặt chẽ để bất đẳng thức không đổi chiều Ta hãy chú ý đến điều kiện bài toán để tìm ước lượng “tốt nhất” Chú ý max{ab, bc, ca} Tìm tài liệu Toán ? Chuyện nhỏ - www.toanmath.com pg 25 (26) max{ab, bc, ca} Tuy nhiên đối với bài toán này thì cần sử dụng điều kiện yếu mà thôi Đầu tiên đưa số nhận xét sau: Đầu tiên ta cần xác định hệ số m để bất đẳng thức trên đúng với x [0,3) Ta thấy trường hợp m nhận bất đẳng thức ngược chiều cho x , tất nhiên đây là điều ta mà không mong muốn Vậy có thể dự đoán m , đó 72m 1 8mx 72m 1 24 48m 1 1 Ta cần có 48m m Vậy nên ta dự đoán m n 48 48 12 Công việc dự đoán đã hoàn tất Bây ta thử chứng minh xem nó có đúng thật không Và ta có bất đẳng thức phụ sau x x 43 ( x 1) (3 x) 0 9 x 48 48(9 x) Điều này hiển nhiên đúng Áp dụng bất đẳng thức phụ trên với các biến ab, bc, ca ta có 1 1 2 43 (a b b c c a 4ab 4bc 4ca) ab bc ac 48 16 Ta cần phải chứng minh bất đẳng thức sau a 2b2 b2c2 c2 a 4ab 4bc 4ca 15 Đặt k ab bc ca , áp dụng bất đẳng thức AM-GM và bất đẳng thức Schur ta có 4x - k 3, abc max 0, Ta xét hai trường hợp sau + Trường hợp Nếu x thì a 2b b 2c c a 4ab 4bc 4ca (ab bc ca )2 4(ab bc ca ) 6abc nhiên đó chưa phải là đánh giá “tốt nhất” vì ta còn có thể làm chặt nữa là k 4k 6abc 81 225 9 15 16 16 + Trường hợp Nếu x thì a 2b b 2c c a 4ab 4bc 4ca (ab bc ca ) 4(ab bc ca ) 6abc k 4k 6abc k 4k 2(4k 9) (k 1)(k 3) 15 15 Bất đẳng thức chứng minh Đẳng thức xảy và a b c Qua quá trình nhận xét và phân tích trên hi vọng các bạn đã hiểu cách tìm hệ số Ở các bài toán sau không thật cần thiết, việc thiết lập bất đẳng thức phụ đưa cách khái quát Chúng ta hãy đến với bài toán sau Bài toán 30 [Moldova TST 2005] Cho a, b, c là các số thực dương thỏa mãn a b c Chứng minh 1 1 ab bc ca Chứng minh Với x , ta luôn có Tìm tài liệu Toán ? Chuyện nhỏ - www.toanmath.com pg 26 (27) x x 12 ( x 1) (3 x) 0 4 x 15 15(4 x) 3 nên ta có 2 3 2(a 2b b c c a ) ab bc ca 36 ab bc ca 15 Áp dụng bất đẳng thức AM-GM và Cauchy-Schawrz ta có a 2b b c c a a b c Lại có max{ab, bc, ca} ab bc ca 3(a 2b b 2c c a ) Cộng các các bất đẳng thức phụ trên vế theo vế ta có điều phải chứng minh Đẳng thức xảy và a b c Nhận xét Đây là bài toán không khó và có nhiều cách tiếp cận khác 1 Áp dụng bất đẳng thức quen thuộc sau và bất đẳng thức AM-GM x y x y Ta có 1 1 1 , a, b [0, 2] 2 2 2 4a b a b 2ab ab Qui bài toán chứng minh 1 1 2 4a 4b c2 Sử dụng bất đẳng thức phụ sau a 15 a2 18 Ngoài ta còn có cách khá trực quan và dễ thực đó là qui đồng và sử dụng bất đẳng thức Schur Bài toán 31 [Phạm Kim Hùng] Cho a, b, c, d là các số thực dương thỏa mãn a b c d Chứng minh 1 1 2 abc bcd cda dab Chứng minh Đây là bài toán khó vì việc thiết lập hệ số phải cần những đánh giá chặt chẽ và suy luận hợp lí Chúng ta hãy cùng phân tích đường đến lời giải bài toán này Ta xác định hệ số m, n cho m( x 1) n( x 1), x0 3 x 3 Như đã phân tích trên ta tìm n 2m , đó bất đẳng thức cần chứng minh tương đương với ( x 1)2 (6m 2mx) Dễ dàng kết luận m đó 16 6m 2mx 6m m 3 Ta cần có Tìm tài liệu Toán ? Chuyện nhỏ - www.toanmath.com pg 27 (28) 16 m0m 16 3 6 3 5 Do nên ta cần có m 16 14 6 5 Từ đây ta chọn m n từ đó ta có bất đẳng thức phụ sau 14 14 x x 12 ( x 1) (8 x) 0 3 x 14 14(3 x) 8 Điều này hiển nhiên đúng với x0 3 6m Sử dụng bất đẳng thức phụ trên và chú ý là max{abc, bcd , cda, dab} 3 suy ta cần chứng minh 5(a 2b2c b2c d c d a d a 2b2 ) 3(abc bcd cda dab) Có thể chứng minh bất đẳng thức trên nhiều cách Sau đây xin trình bày cách dựa vào kỹ thuật hàm lồi a b2 c d ,k , x ab, y cd đó, ta có t x, k y Bất đẳng thức Đặt t 2 cần chứng minh tương đương với f ( x) 10 x k 10 y 2t 3x y 2k y x 2t Ta có f // ( x) 20k 3y (2 x 2t )3 0 Suy f ( x) là hàm lồi, đó f ( x) max{ f (t ), f (0)} Ta có f (0) yt yt yt 3 f (t ) 10 y 2t yt 10k 2t 3t y 2k g ( y ) Tương tự trên ta có g ( y ) là hàm lồi nên g ( y) max{g (k ), g (0)} Ta có g (0) kt 5kt kt 3 k t2 1 g (t ) 4(kt 1)(5kt 1) kt Vậy bất đẳng thức chứng minh xong Ngay từ ban đầu chúng tôi đã nói đây là bất đẳng thức không dễ và đòi hỏi những đánh giá chặt chẽ U.C.T đây đóng vai trò là bàn đạp quan trọng để đến lời giải Tìm tài liệu Toán ? Chuyện nhỏ - www.toanmath.com pg 28 (29) Bài toán 32 [Võ Quốc Bá Cẩn] Cho các số thực a, b, c, d thỏa a b2 c d , chứng minh bất đẳng thức 1 1 16 ab bc cd da Chứng minh Tương tự các bài toán trước, ta thiết lập bất đẳng thức sau với x 32 x 10 1 x Từ đây, ta suy 1 1 32 40 ( a 2b b c c d d a ) ab bc cd da 9 32 40 (a c )(b d ) 9 40 16 (a b c d )2 9 Từ đây, ta có đpcm Đẳng thức xảy và a b c d Nhận xét Bài toán này đặt để “làm mạnh” bài toán sau Phạm Văn Thuận 1 1 1 8 ab bc cd da ac bd với cùng giả thiết trên Lời giải tác giả cho bài toán này dài và phức tạp, dùng U.C.T mở rộng ta lại có lời giải ngắn gọn và đơn giản! Ngoài ra, chúng ta còn có cách “làm mạnh” khác cho bài toán Phạm Văn Thuận, ta có 1 1 1 8 2 2 2 ab bc cd d a ac bd 1 1 1 1 1 1 Bài toán này đã bạn ZhaoBin, sinh viên người Trung Quốc đưa lời giải đẹp cách sử dụng bất đẳng thức Cauchy-Schwarz, đây, chúng tôi xin giới thiệu lời giải khác theo tư tưởng U.C.T Sử dụng bất đẳng thức bài, ta cần chứng minh (a b)4 (a c)4 (a d )4 (b c)4 (b d )4 (c d )4 Thật vậy, ta có (a b)4 2(a b4 6a 2b2 ) (a b)4 2(a b4 6a 2b2 ) Tương tự với các số hạng còn lại, ta suy VT 6(a b2 c d )2 Bất đẳng thức chứng minh xong Thật tự nhiên, câu hỏi sau đặt ra, liệu bất đẳng thức sau có đúng? 1 1 16 2 2 ab bc cd d a 1 1 1 1 Thật đáng tiếc là bất đẳng thức trên lại không đúng! Các bạn cần cho a b 0.4, và c d 0.84 Tìm tài liệu Toán ? Chuyện nhỏ - www.toanmath.com pg 29 (30) Bài toán 33 [Vasile Cirtoaje] Cho các số không âm a, b, c, d có tổng 4, chứng minh bất đẳng thức sau 1 1 1 abc bcd cda dab Chứng minh Ta có thể thiết lập bất đẳng thức sau với x 48 x x 10 5 x Do đó +, Nếu max{abc, bcd , cda, dab} thì sử dụng bất đẳng thức trên, ta cần chứng minh a 2b2c2 b2c2 d c d a d 2a 2b2 abc bcd cda dab Bất đẳng thức này có thể dễ dàng chứng minh dồn biến dùng hàm lồi +, Nếu max{abc, bcd , cda, dab} , không tính tổng quát, giả sử abc , đó, chú ý với x, y 0, x y 5, ta có 1 1 xy (10 x y ) 0 5 x y x y 5(5 x)(5 y )(5 x y ) Suy 1 1 5 x 5 y 5 x y Và đó, với x, y, z 0, x y z ta có 1 1 5 x 5 y 5 z 5 x y z 2 Chú ý (a b c abc) và abc nên bcd cda dab , đó cyc 1 1 bcd cda dab d (ab bc ca ) Ta cần chứng minh 1 abc d (ab bc ca) Đặt x d , abc nên x a b c 3 abc 3 , theo bất đẳng thức AMGM thì abc x , ab bc ca x 27 Do đó, ta cần chứng minh 1 1 x3 x (4 x) 27 Hay f ( x) x6 x5 80 x3 360 x 675 Ta có f / ( x) x ( x 4) x3 ( x 12) 48 x(15 x) Suy f ( x) là hàm nghịch biến, đó f ( x) f (3 2) 27(48 77) Từ đây, ta có đpcm Đẳng thức xảy và a b c d Tìm tài liệu Toán ? Chuyện nhỏ - www.toanmath.com pg 30 (31) Phần Lời kết Sau quá trình tìm hiểu và phân tích cụ thể các bài toán, hẳn các bạn đã phần nào cảm nhận nét đẹp U.C.T dù thực đây là kĩ thuật cực kì đơn giản và dễ hiểu Chúng tôi không xem U.C.T là phương pháp chính thống mà đơn giản nó là kĩ thuật cần biết và cần nắm vững bạn học bất đẳng thức Nhiều người quan niệm U.C.T không có ý nghĩa gì theo thân chúng tôi nó nên khái quát để sử dụng số trường hợp U.C.T là bước đệm quan trọng và đôi mang nhiều ý nghĩa trên đường tìm lời giải cho bài toán Một kĩ thuật hay không thiết nó là nó giải tất các dạng toán mà là nó phải đưa ta đến những ý tưởng, đường sáng sủa, dễ nghĩ, dễ nhận thấy mặt trực quan Trong chuyên đề này nhiều bài toán hình thức cồng kềnh USAMO 2003, JMO 1997, là những bài toán không khó, không chọn đúng hướng làm thì dẫn đến những lời giải chấp nhận đúng mặt toán học Đó là những bài toán đại diện cho U.C.T kết hợp với kĩ thuật chuẩn hóa Tuy nhiên đó chưa phải là điểm dừng Ở phần tiếp theo, xuất nhiều bài toán mang đậm sắc tức là sử dụng mỗi U.C.T thì không đích Cách khắc phục là phân chia trường hợp để giải Đây chính là sở để tìm các khoảng nghiệm cần xét biến Việc đánh giá đây đòi hỏi người làm tinh tế và khéo léo các phần trước Tuy nhiên bạn có niềm tin chuyện có thể giải Sau đã nắm tay những kiến thức định kỹ thuật chúng ta bước qua khoảng không gian phức tạp đó là dùng U.C.T để giải những bài toán mà điểm cực trị đạt chỗ Bao gồm trường hợp đó là tất các biến và trường hợp có (n 1) biến khác biến còn lại Ở đây ta chú ý đến bất đẳng thức Vornicu Schur để khắc phục nhược điểm U.C.T Phần kĩ thuật phân tách theo tổng là dạng đẹp kỹ thuật này, số bài toán tiêu biểu cho dạng phân tách này là IMO 2001 và số bài toán đã nêu trên Dù U.C.T đây dùng theo tư tưởng khác với các phần trước Như các bạn đã biết U.C.T thông thường biết đến với các bài toán mà biến số độc lập không liên quan đến Tuy nhiên xét với lớp bài toán thì chưa lột tả hết nét đẹp kĩ thuật đơn giản này Ta có thể sử dụng U.C.T để tìm những bất đẳng thức phụ với điều kiện liên quan mật thiết với Tức là không tách theo đơn lượng biến để giải U.C.T đây đòi hỏi bạn phải có những kiến thức hàm số để tìm các ước lượng chinh xác Cuối cùng chúng ta đã đến số bài toán khó mà theo nhiều người quan niệm là không thể giải U.C.T, điều này là điểm yếu kỹ thuật này Khi việc thiết lập hệ số thắt chặt thì chuyện khác Như các bạn đã thấy trên U.C.T mở rộng mang những đặc điểm phức tạp hiệu mang lại thì là bất ngờ Một số bài toán khó đã đưa dạng đơn giản để giải theo số phương pháp đã biết Đó là nét khá độc đáo kỹ thuật này Tuy nhiên hẳn đó chưa phải là U.C.T “chặt” Còn nhiều điều nữa kỹ thuật này chờ các bạn khám phá Chúng tôi xin kết thúc bài viết này đây Hi vọng với những dòng tâm cùng các bạn bất đẳng thức đã phần nào gợi mở cho các bạn cái gì đó giúp các bạn tìm thêm những ý tưởng sáng tạo mới, những hiểu biết Và hãy luôn quan niệm đằng sau lời giải cho mỗi bài toán là quá trình dự đoán, thử, sai và đúng Hẹn gặp lại các bạn ngày không xa Tìm tài liệu Toán ? Chuyện nhỏ - www.toanmath.com pg 31 (32) Phần 10 Bài tập áp dụng Bài toán [Diễn đàn toán học] Cho a, b, c, d , e là các số thực không âm thỏa mãn a3 b3 c3 d e3 Tìm giá trị nhỏ biểu thức a2 b2 c2 d2 e2 a b c d e3 Bài toán [Vasile Cirtoaje, Crux Mathematicorum, Problem 3032] Cho a, b, c là các số thực không âm thỏa mãn a b c2 Chứng minh 1 ab bc ca Bài toán [Mathematical Excalibur, Vol 9, Num 1, 8/2004] Cho a, b, c, d là các số thực dương thỏa mãn a b c d Chứng minh 6(a b c d ) (a b c d ) Bài toán [Mihai Piticari, Dan Popescu, Old and New Inequalities] Cho a, b, c là các số thực dương nhỏ thỏa mãn a b c Chứng minh 6(a b c ) 5(a b c ) Bài toán [Titu Andreescu, Gabriel Dospinescu, Old and New Inequalities] Cho a, b, c là các số thực dương nhỏ thỏa mãn a b c Chứng minh 1 27 2 1 a 1 b 1 c 10 Bài toán [Andrian Zahariuc, Old and New Inequalities] Cho a, b, c (1, 2) Chứng minh b a c b a c 1 4b c c a 4c a a b 4a b b c Bài toán [Vũ Đình Quý] Cho a, b, c là các số thực dương thỏa mãn abc Chứng minh 1 3 a a 1 b b 1 c c 1 Bài toán [Vasile Cirtoaje] Cho a, b, c, d là các số thực dương thỏa mãn abcd Chứng minh 1 a 1 b 1 c 1 d 4 a b2 c d Bài toán [Vasile Cirtoaje, GM-B,11,1999] Cho a, b, c, d là các số thực dương thỏa mãn abcd Chứng minh Tìm tài liệu Toán ? Chuyện nhỏ - www.toanmath.com pg 32 (33) 1 1 1 3 1 a a a 1 b b b 1 c c c 1 d d d Bài toán 10 [China TST 2004] Cho a, b, c, d là các số thực dương thỏa mãn abcd Chứng minh 1 1 1 2 (1 a) (1 b) (1 c) (1 d ) Bài toán 11 [Arkady Alt, Crux mathematicorum] Chứng minh với a, b, c ta có a b c a / b2 / c2 / ab bc ca Complete Trong Võ Quốc Bá Cẩn Nguyễn Thúc Vũ Hoàng bài viết có sử dụng nhiều bài toán trích dẫn từ Algebraic Inequalites – Old and New Method tác giả Vasile Cirtoaje Sáng tạo Bất đẳng thức tác giả Phạm Kim Hùng Old and New Inequalities các tác giả Titu Andreescu, Vasile Cirtoaje, G Dospinescu, M Lascu Tìm tài liệu Toán ? Chuyện nhỏ - www.toanmath.com pg 33 (34)